Đến nội dung

Hình ảnh

BĐT AM-GM


  • Please log in to reply
Chủ đề này có 339 trả lời

#281
Element hero Neos

Element hero Neos

    Trung úy

  • Thành viên
  • 943 Bài viết

Bạn chú ý điều kiện là $0<x<1$ nên bạn giải sai rồi.

nếu x càng sát mốc 1 và 0 thì A càng nhỏ nên không tìm được min



#282
Cuongpa

Cuongpa

    Thượng sĩ

  • Thành viên
  • 238 Bài viết

Có min đó bạn ạ.  $Min=\frac{2}{3\sqrt{3}}\Leftrightarrow x=\frac{1}{\sqrt{3}}$


Bài viết đã được chỉnh sửa nội dung bởi Cuongpa: 18-11-2015 - 21:30

Success doesn't come to you. You come to it.


#283
tpdtthltvp

tpdtthltvp

    Trung úy

  • Điều hành viên THCS
  • 831 Bài viết

Bạn đưa ra lời giải đi!


$\color{red}{\mathrm{\text{How I wish I could recollect, of circle roud}}}$

$\color{red}{\mathrm{\text{The exact relation Archimede unwound ! }}}$

 


#284
royal1534

royal1534

    Trung úy

  • Điều hành viên THCS
  • 773 Bài viết
 

Tìm GTNN của biểu thức sau khi $0<x<1$:

            $A=x(1-x^{2})$

 
 

Bạn đưa ra lời giải đi!

Áp dụng bất đẳng thức AM-GM ta có $2A^{2}=2x^{2}(1-x^{2})^{2}=2x^{2}.(1-x^{2}).(1-x^{2}) \leq \frac{(2x^{2}+1+1-x^{2}-x^{2})^{3}}{27}=\frac{8}{27}$

$\rightarrow A \leq \sqrt{\frac{4}{27}}=\frac{2.\sqrt{3}}{9} $

Dấu '=' xảy ra khi $2x^{2}=1-x^{2}$ 

                   $\Leftrightarrow 3x^{2}=1$

                   $\Leftrightarrow x=\frac{1}{\sqrt{3}}$


Bài viết đã được chỉnh sửa nội dung bởi royal1534: 18-11-2015 - 21:11


#285
Element hero Neos

Element hero Neos

    Trung úy

  • Thành viên
  • 943 Bài viết

 

 
 
 

Áp dụng bất đẳng thức AM-GM ta có $2A^{2}=2x^{2}(1-x^{2})^{2}=2x^{2}.(1-x^{2}).(1-x^{2}) \leq \frac{(2x^{2}+1+1-x^{2}-x^{2})^{3}}{27}=\frac{8}{27}$

$\rightarrow A \leq \sqrt{\frac{4}{27}}=\frac{2.\sqrt{3}}{9} $

Dấu '=' xảy ra khi $2x^{2}=1-x^{2}$ 

                   $\Leftrightarrow 3x^{2}=1$

                   $\Leftrightarrow x=\frac{1}{\sqrt{3}}$

 

đấy là max chứ có phải min đâu bạn! :(



#286
royal1534

royal1534

    Trung úy

  • Điều hành viên THCS
  • 773 Bài viết

đấy là max chứ có phải min đâu bạn! :(

Mình nghĩ nó không có Min đâu



#287
Element hero Neos

Element hero Neos

    Trung úy

  • Thành viên
  • 943 Bài viết

Mình nghĩ nó không có Min đâu

 

nếu x càng sát mốc 1 và 0 thì A càng nhỏ nên không tìm được min

Cắc chắn ko có vì như mình nói



#288
NguyenPhuongQuynh

NguyenPhuongQuynh

    Hạ sĩ

  • Thành viên
  • 87 Bài viết

Các bạn giúp mình bài này nhé!

$Cho \left\{x,y,z>0\begin{matrix} \\ xyz=1\end{matrix}\right.$$CMR:\frac{1}{x^{3}(y+z)}+\frac{1}{y^{3}(z+x)}+\frac{1}{z^{3}(x+y)}\geq \frac{3}{2}$



#289
tpdtthltvp

tpdtthltvp

    Trung úy

  • Điều hành viên THCS
  • 831 Bài viết

Ta có:

VT=$\sum \frac{\frac{1}{x^2}}{x(y+z)}\geq \frac{(\sum \frac{1}{x})^2}{2\sum xy}=\frac{\sum \frac{1}{x}}{2}\geq \frac{3\frac{1}{\sqrt[3]{xyz}}}{2}=\frac{3}{2}(dpcm)$


$\color{red}{\mathrm{\text{How I wish I could recollect, of circle roud}}}$

$\color{red}{\mathrm{\text{The exact relation Archimede unwound ! }}}$

 


#290
Element hero Neos

Element hero Neos

    Trung úy

  • Thành viên
  • 943 Bài viết

Các bạn giúp mình bài này nhé!

$Cho \left\{x,y,z>0\begin{matrix} \\ xyz=1\end{matrix}\right.$$CMR:\frac{1}{x^{3}(y+z)}+\frac{1}{y^{3}(z+x)}+\frac{1}{z^{3}(x+y)}\geq \frac{3}{2}$

đây



#291
NguyenPhuongQuynh

NguyenPhuongQuynh

    Hạ sĩ

  • Thành viên
  • 87 Bài viết

Ta có:

VT=$\sum \frac{\frac{1}{x^2}}{x(y+z)}\geq \frac{(\sum \frac{1}{x})^2}{2\sum xy}=\frac{\sum \frac{1}{x}}{2}\geq \frac{3\frac{1}{\sqrt[3]{xyz}}}{2}=\frac{3}{2}(dpcm)$

mình chưa học kí hiệu tổng đó nên bạn viết rõ ra được không? Với lại mình không hiểu bước biến đổi đầu tiên, có phải bạn áp dụng :1/a+1/b+1/c>= 9/(a+b+c) k? 



#292
Element hero Neos

Element hero Neos

    Trung úy

  • Thành viên
  • 943 Bài viết

mình chưa học kí hiệu tổng đó nên bạn viết rõ ra được không? Với lại mình không hiểu bước biến đổi đầu tiên, có phải bạn áp dụng :1/a+1/b+1/c>= 9/(a+b+c) k? 

bạn có thể xem tổng đó(gọi là tổng xích-ma) tại đây nhé!



#293
tpdtthltvp

tpdtthltvp

    Trung úy

  • Điều hành viên THCS
  • 831 Bài viết

Mình cũng chưa học đâu, mình học được từ diễn đàn đó :lol: .Chi tiết:

VD:theo như trên, thì $\sum \frac{\frac{1}{x^2}}{x(y+z)}=\frac{\frac{1}{x^2}}{x(y+z)}+\frac{\frac{1}{y^2}}{y(x+z)}+\frac{\frac{1}{z^2}}{z(x+y)}$

Áp dụng bđt xvác bạn ạ.


$\color{red}{\mathrm{\text{How I wish I could recollect, of circle roud}}}$

$\color{red}{\mathrm{\text{The exact relation Archimede unwound ! }}}$

 


#294
Element hero Neos

Element hero Neos

    Trung úy

  • Thành viên
  • 943 Bài viết

tiếp đây  $\sum \sqrt[3]{\frac{a^{2}+bc}{b^{2}+c^{2}}}\geq 9.\frac{\sqrt[3]{abc}}{a+b+c}$



#295
NguyenPhuongQuynh

NguyenPhuongQuynh

    Hạ sĩ

  • Thành viên
  • 87 Bài viết

Mình cũng chưa học đâu, mình học được từ diễn đàn đó :lol: .Chi tiết:

VD:theo như trên, thì $\sum \frac{\frac{1}{x^2}}{x(y+z)}=\frac{\frac{1}{x^2}}{x(y+z)}+\frac{\frac{1}{y^2}}{y(x+z)}+\frac{\frac{1}{z^2}}{z(x+y)}$

Áp dụng bđt xvác bạn ạ.

Cám ơn bạn nhiều lắm ! Nhưng mà mình học chỉ được áp dụng Cô-si còn các bất đẳng thức khác phải tự chứng minh mới được sử dụng. Bạn có cách nào dùng Cô-si không?



#296
Element hero Neos

Element hero Neos

    Trung úy

  • Thành viên
  • 943 Bài viết

tiếp bài nữa về tổng xich-ma: $\sum \frac{a}{b}\geq \sum \sqrt{\frac{a^{2}+1}{b^{2}+1}}$ (với a,b,c là các số thực)



#297
NguyenPhuongQuynh

NguyenPhuongQuynh

    Hạ sĩ

  • Thành viên
  • 87 Bài viết

tiếp đây  $\sum \sqrt[3]{\frac{a^{2}+bc}{b^{2}+c^{2}}}\geq 9.\frac{\sqrt[3]{abc}}{a+b+c}$

mình không hiểu, đây là một bất đẳng thức khác hay bạn trả lời câu hỏi vừa nãy của mình



#298
Element hero Neos

Element hero Neos

    Trung úy

  • Thành viên
  • 943 Bài viết

mình không hiểu, đây là một bất đẳng thức khác hay bạn trả lời câu hỏi vừa nãy của mình

Chẳng phải mình bảo đây là bài tiếp sao, mình đưa ra cho các bạn làm.


Bài viết đã được chỉnh sửa nội dung bởi Element hero Neos: 01-12-2015 - 20:24


#299
phamhuy1801

phamhuy1801

    Trung sĩ

  • Thành viên
  • 181 Bài viết

Cám ơn bạn nhiều lắm ! Nhưng mà mình học chỉ được áp dụng Cô-si còn các bất đẳng thức khác phải tự chứng minh mới được sử dụng. Bạn có cách nào dùng Cô-si không?

 

Nếu chỉ được dùng $Cauchy$.

Bạn có thể đặt $a=\frac{1}{x}; b=\frac{1}{y}; c=\frac{1}{z}$

$BĐT$ đã cho tương đương với $\frac{a^2}{b+c}+\frac{b^2}{c+a}+\frac{c^2}{a+b} \ge \frac{3}{2}$

Dùng $Cauchy$: $\frac{a^2}{b+c} + \frac{b+c}{4} \ge a$

Thiết lập các $BĐT$ tương tự rồi cộng theo vế: $\frac{a^2}{b+c}+\frac{b^2}{c+a}+\frac{c^2}{a+b} \ge \frac{a+b+c}{2} \ge \frac{3}{2}$ (dùng $Cauchy$ 3 số và sử dụng $abc=1$)



#300
NguyenPhuongQuynh

NguyenPhuongQuynh

    Hạ sĩ

  • Thành viên
  • 87 Bài viết

$Cho \left\{x,y,z>0\begin{matrix} \\xyz=1 \end{matrix}\right. CMR:\frac{x^{2}}{x+y+y^{3}z}+\frac{y^{2}}{y+z+z^{3}x}+\frac{z^{2}}{x+z+x^{3}y} \geq 1$


Bài viết đã được chỉnh sửa nội dung bởi NguyenPhuongQuynh: 02-12-2015 - 20:37





2 người đang xem chủ đề

0 thành viên, 2 khách, 0 thành viên ẩn danh